Difference between revisions of "2022 AMC 10B Problems/Problem 7"

Line 2: Line 2:
  
 
<math>p+q=-k</math>
 
<math>p+q=-k</math>
 +
 
<math>p*q=36</math>
 
<math>p*q=36</math>
 +
 
(Let <math>p</math> and <math>q</math> be the roots)
 
(Let <math>p</math> and <math>q</math> be the roots)
  
Line 9: Line 11:
 
We cancel out the <math>6</math> and <math>6</math> because the problem states that it wants distinct roots.
 
We cancel out the <math>6</math> and <math>6</math> because the problem states that it wants distinct roots.
  
Thus, we have a total of <math>4</math> pairs and another <math>4</math> pairs (the negatives), which total us <math>4+4=8</math>.
+
Thus, we have a total of <math>4</math> pairs and another <math>4</math> pairs (the negatives), which total us <math>4+4=\boxed{\textbf{(B) }8}</math>.
<math>\boxed{\textbf{(B) }8\boxed</math>.
 

Revision as of 16:27, 17 November 2022

Using Vieta's Formula, this states:

$p+q=-k$

$p*q=36$

(Let $p$ and $q$ be the roots)

This shows that p and q must be the factors of $36$: $1, 36, 2, 18, 3, 12, 4, 9, 6$ and its negative counterpart.

We cancel out the $6$ and $6$ because the problem states that it wants distinct roots.

Thus, we have a total of $4$ pairs and another $4$ pairs (the negatives), which total us $4+4=\boxed{\textbf{(B) }8}$.